viernes, 19 de diciembre de 2014

Ejemplo de inducción matemática en trigonometría .

 Nota.Por cuestiones de tipografía, la función $sen\theta$, va a  aparecer como   $ \sin\theta$ 


  Ejercicio 11. Demostrar:   $\sin\theta+\sin2\theta+\cdots+\sin n\theta= \frac{\sin\frac{1}{2}\left(n+1\right)\theta \sin\frac{1}{2} n\theta}{\sin\frac{\theta}{2}}$  

Demostración:

Para:
         $n=1$, tenemos
                                 $\sin 1\cdot \theta = \frac{\sin\frac{1}{2}\left(1+1\right)\theta \sin\frac{1}{2} 1       \cdot \theta}{\sin\frac{\theta}{2}}$

                                   $\sin 1\cdot \theta = \sin\frac{1}{2}\left(2\right)\theta$

                                  $\sin \theta=\sin \theta$     $\checkmark$

       Aunque no se pide revisamos para $n=2$,

                               $\sin 1\cdot \theta+\sin 2\cdot \theta = \sin\theta+\sin 2\theta$
                                                               $ =\sin \theta+2\sin\theta \cos\theta$
                                                               $=\left(1+2\cos \theta\right)\sin \theta$
                                                              $=\left(3-2+2\cos \theta\right)\sin \theta$
                                                              $=\left[3-2\left(1-\cos \theta\right)\right]\sin \theta$
                                                              $=\left[3-4\frac{\left(1-\cos \theta\right)}{2}\right]\sin \theta$
                                                              $=\left[3-4\sin^2\frac{\theta}{2}\right]\sin \theta$

                                                            $ =\frac{\sin\frac{\theta}{2}\left[3-4\sin^2\frac{\theta}{2}\right]\sin \theta}{\sin \frac{\theta}{2}}$

                                                           $ =\frac{\left(3\sin\frac{\theta}{2}-4\sin^3\frac{\theta}{2}\right)\sin\theta}{\sin \frac{\theta}{2}}$


                                                          $ =\frac{\sin\frac{3\theta}{2} \sin \theta}{\sin \frac{\theta}{2}}.$         $\checkmark$


HI       $\sin\theta+\sin2\theta+\cdots+\sin k\theta= \frac{\sin\frac{1}{2}\left(k+1\right)\theta \sin\frac{1}{2} k\theta}{\sin\frac{\theta}{2}}.$                            $\checkmark$


TI       $\sin\theta+\sin2\theta+\cdots+\sin k^*\theta= \frac{\sin\frac{1}{2}\left(k^*+1\right)\theta \sin\frac{1}{2} k^*\theta}{\sin\frac{\theta}{2}}.$


Prueba:
            $\sin\theta+\sin2\theta+\cdots+\sin k^*\theta=\sin\theta+\sin2\theta+\cdots+\sin k\theta+\sin k^*\theta$

                                                                             $= \frac{\sin\frac{1}{2}\left(k+1\right)\theta \sin\frac{1}{2} k\theta}{\sin\frac{\theta}{2}}+\sin\left( k+1\right)\theta$
                        
    $= \frac{\sin\frac{1}{2}\left(k+1\right)\theta \sin\frac{1}{2} k\theta}{\sin\frac{\theta}{2}}+\sin2\left(\frac{1}{2}\left( k+1\right)\right)\theta$

    $=\frac{\sin\frac{1}{2}\left(k+1\right)\theta \sin\frac{1}{2} k\theta}{\sin\frac{\theta}{2}}+2\sin\frac{1}{2}\left(k+1\right)\theta\cos\frac{1}{2}\left(k+1\right)\theta$

 $=\frac{\sin\frac{1}{2}\left(k+1\right)\theta \sin\frac{1}{2} k\theta+2\sin\frac{\theta}{2}\sin\frac{1}{2}\left(k+1\right)\theta\cos\frac{1}{2}\left(k+1\right)\theta}{\sin\frac{\theta}{2}}$

$=\frac{\sin\frac{1}{2}\left(k+1\right)\theta}{\sin\frac{\theta}{2}}\left[\sin\frac{1}{2} k\theta +2\sin\frac{\theta}{2}\cos \frac{\left(k+1\right)\theta}{2} \right]$
 $=\frac{\sin\frac{1}{2}\left(k+1\right)\theta}{\sin\frac{\theta}{2}}\left[\sin\frac{1}{2} k\theta +2\sin\frac{\theta}{2}\cos {\left(\frac{k\theta}{2}+\frac{\theta}{2}\right)} \right]$                                        $=\frac{\sin\frac{1}{2}\left(k+1\right)\theta}{\sin\frac{\theta}{2}}\left[\sin\frac{1}{2} k\theta +2\sin\frac{\theta}{2}\left(\cos \frac{k\theta}{2}\cos\frac{\theta}{2}-\sin\frac{k\theta}{2}\sin\frac{\theta}{2}\right)\right]$

$=\frac{\sin\frac{1}{2}\left(k+1\right)\theta}{\sin\frac{\theta}{2}}\left[\sin\frac{1}{2} k\theta +2\sin\frac{\theta}{2}\cos \frac{k\theta}{2}\cos\frac{\theta}{2}-2\sin\frac{\theta}{2}\sin\frac{k\theta}{2}\sin\frac{\theta}{2}\right]$
 
$=\frac{\sin\frac{1}{2}\left(k+1\right)\theta}{\sin\frac{\theta}{2}}\left[\sin\frac{1}{2} k\theta\left(1-2\sin^2\frac{\theta}{2}\right) +\cos \frac{k\theta}{2}2\sin\frac{\theta}{2}\cos\frac{\theta}{2}\right]$

$=\frac{\sin\frac{1}{2}\left(k+1\right)\theta}{\sin\frac{\theta}{2}}\left[\sin\frac{1}{2} k\theta\cos\theta +\cos \frac{k\theta}{2}\sin\theta\right]$

$=\frac{\sin\frac{1}{2}\left(k+1\right)\theta}{\sin\frac{\theta}{2}}\sin\left(\frac{k\theta}{2}+\theta\right)$

$=\frac{\sin\frac{1}{2}\left(k+1\right)\theta}{\sin\frac{\theta}{2}}\sin\left(\frac{k\theta+2\theta}{2}\right)$

$=\frac{\sin\frac{1}{2}\left(k+1\right)\theta\sin\left(\frac{k\theta+2\theta}{2}\right)}{\sin\frac{\theta}{2}}$

$=\frac{\sin\frac{1}{2}\left(k+1\right)\theta\sin\left(\frac{\left(k+2\right)\theta}{2}\right)}{\sin\frac{\theta}{2}}$

$=\frac{\sin\frac{1}{2}\left(k+1\right)\theta\sin\left[\frac{\left(\left(k+1\right)+1\right)\theta}{2}\right]}{\sin\frac{\theta}{2}}$

$=\frac{\sin\frac{1}{2}k^*\theta\sin\left[\frac{\left(k^*+1\right)\theta}{2}\right]}{\sin\frac{\theta}{2}}.$   $\checkmark$
 

























































































martes, 9 de diciembre de 2014

Ejercicios de inducción matemática III

En esta entrada se revisan las leyes de los exponentes para números naturales, que en esencia son las mismas que se aplican a los números reales. Dominar estas leyes proporciona una herramienta algebraica inigualable para estudios superiores. Todos los números y todas las variables que aparecen en esta entrada pertenecen al conjunto de los números naturales  $\mathbb{N}=\{1,2,3,...,\}.$

Definición:         $m^1=m$,        $m^{p+1}=m^p\cdot m$


Ejercicio 8.  Demostrar:       $m^p\cdot m^q=m^{p+q}$.

Demostración: Sea  $P(q):m^p\cdot m^q.$

Entonces:    $P(1): m^p\cdot m^1=m^p\cdot m=m^{p+1}$,               $\checkmark$

HI                                             $m^p\cdot m^k=m^{p+k}$,                                        $\checkmark$

TI                                    $m^p\cdot m^{k^*}=m^{p+k^*}.$
Prueba:         $m^p\cdot m^{k^*}=m^p \cdot m^{k+1}$
                                                             $=m^p \cdot \left(m^k \cdot m^1\right)$
                                                             $=\left(m^p \cdot m^k\right)\cdot m^1$
                                                             $=\left(m^{p+k}\right)\cdot m^1$
                                                             $=m^{\left(p+k\right)+1}$
                                                             $=m^{p+k^*}$.                       $\checkmark$


Ejercicio 9.   Demostrar:              $\left(m^p\right)^q=m^{p\cdot q}$.

Demostración: Sea                         $P(q): \left(m^p\right)^q$.
Entonces:                                     $P(1):\left(m^p\right)^1=m^p=m^{p\cdot 1}.$             $\checkmark$

HI                                               $P(k):\left(m^p\right)^k=m^{p\cdot k}$.                              $\checkmark$

TI                                              $P(k^*):\left(m^p\right)^{k^*}=m^{p\cdot k^*}$.
Prueba:                                      $\left(m^p\right)^{k^*}=\left(m^p\right)^{k+1}$
                                                              $=\left(m^p\right)^k\cdot (m^p)$
                                                              $=m^{p\cdot k}\cdot m^p$
                                                              $=m^{p\cdot k+p}$
                                                              $=m^{p\cdot k^*}.$                               $\checkmark$


Ejercicio 10. Demostrar:     $(m \cdot n )^p=m^p \cdot n^p.$

Demostración: Sea     $P(p):(m \cdot n )^p=m^p \cdot n^p.$

Entonces:                 $P(1):(m \cdot n )^1=m \cdot n=m^1 \cdot n^1$               $\checkmark$

HI                          $P(k):(m \cdot n )^k=m^k \cdot n^k$                              $\checkmark$

TI                          $P(k^*):(m \cdot n )^{k^*}=m^{k^*} \cdot n^{k^*}$

Prueba:                  $(m \cdot n )^{k^*}=(m \cdot n )^{k+1}$
                                             $=(m \cdot n )^k \cdot (m \cdot n)^1$
                                             $=(m^k \cdot n ^k) \cdot (m^1 \cdot n^1)$
                                             $=(m^k \cdot m ^1) \cdot (n^k \cdot n^1)$
                                             $=m^{k +1} \cdot n^{k +1}$
                                             $=m^{k ^*} \cdot n^{k ^*}.$                      $\checkmark$

Ejercicio 11. Demostrar:      $(1)^p=1$.

Demostración:  Sea  $P(p):(1)^p.$

Entonces:         $P(1):(1)^1=1$                $\checkmark$

HI                  $P(k):(1)^k=1$                $\checkmark$

TI                  $P(k^*):(1)^{k^*}=1$
Prueba:          $(1)^{k^*}=(1)^{k+1}$
                              $=1^k\cdot1^1$
                              $=1\cdot1$
                              $=1$.


Nótese la utilización reiterada de lo que se ha ido definiendo en el desarrollo de cada problema. Cómo vamos usando lo ya probado o supuesto verdadero conforme llevamos adelante el razonamiento. Revise con atención cada paso y se dará cuenta de que al avanzar siempre utilizamos una idea anterior de éste o de anteriores ejercicios. Esa es la ventaja y el poder del método, que no necesitamos probar una y otra vez cada problema, sino que podemos utilizar con confianza y precisión lo ya probado para construir un conocimiento sólido que nos permita adentrarnos en las Ciencias Exactas,
















































































jueves, 4 de diciembre de 2014

Ejercicios de inducción matemática II

Continuamos resolviendo problemas de inducción matemática, con $\mathbb{N}= \{1,2,3,...\}. $

Ejercicio 4. Demostrar,  para $a$, $d$ fijos y $n$ variable

                          $a+(a+d)+(a+2d)+\cdots+\left(a+(n-1)d \right)= \frac{n}{2}\left(2a+(n-1)d \right).$

Demostración:

Para:   $n=1$,         $a+(1-1)d= \frac{1}{2}\left(2a+(1-1)d \right)$
                             
                                           $a=a$.                                                      $\checkmark$  

          $n=k$,
  HI        $a+(a+d)+(a+2d)+\cdots+\left(a+(k-1)d \right)= \frac{k}{2}\left(2a+(k-1)d \right).$ $\checkmark$


         $n=k^*$,
TI     $a+(a+d)+(a+2d)+\cdots+\left(a+(k^*-1)d \right)=\underbrace{a+(a+d)+(a+2d)+\cdots+\left(a+(k-1)d \right)}+\left(a+(k^*-1)d \right)$                                                                                                                           HI


                                                                          $=\underbrace{\frac{k}{2}\left(2a+(k-1)d \right)}+\left(a+(k^*-1)d \right)$                                                                                            HI


                                                  $=\frac{k\left(2a+(k-1)d \right)+2\left(a+(k^*-1)d \right)}{2}$

                                                  $=\frac{k\left(2a+(k-1)d \right)+2\left(a+\left((k+1)-1\right)d \right)}{2}$

                                                  $=\frac{k\left(2a+kd-d \right)+2\left(a+kd \right)}{2}$
                                                                             
                                                   $=\frac{k2a+k^2d-kd +2a+2kd }{2}$

                                                  $=\frac{k2a+2a+k^2d+kd }{2}$

                                                  $=\frac{\left(k+1\right)2a+kd\left(k+1\right) }{2}$

                                                  $=\frac{\left(k+1\right)\left(2a+kd\right) }{2}$
                                                                        $=\frac{\left(k+1\right)\left[2a+\left(\left(k+1\right)-1\right)d\right] }{2}$

                                                  $=\frac{k^* \left[2a+\left(k^*-1\right)\right]d}{2}$      $\checkmark$



Ejercicio 5. Demostrar       $2+2^2+2^3+\cdots2^n=2\left(2^n-1\right).$

Para:  $n=1$,             $2^1=2\left(2^1-1\right)=2\left(2-1\right)=2\cdot 1=2.$            $\checkmark$

 HI   $n=k$,    $2+2^2+2^3+\cdots2^k=2\left(2^k-1\right).$  $\checkmark$                                                                                 

TI    $n=k^*$          $2+2^2+2^3+\cdots2^{k^*}=2\left(2^{k^*}-1\right).$                  


Prueba:             $2+2^2+2^3+\cdots2^{k^*}=\underbrace{2+2^2+2^3+\cdots2^k}+2^{k^*}$
                                                               HI
                                                                     $=\underbrace{2\left(2^k-1\right)}+2^{k^*}$
                                                                                  HI
                                                                     $=2^{k+1}-2+2^{k+1}$

                                                                     $=2\cdot 2^{k+1}-2$
                                                                 
                                                                     $=2\left( 2^{k^*}-1\right).$           $\checkmark$

Ejercicio 6.  Demostrar:

   $1+\frac{1}{2}+\frac{1}{2^2}+\frac{1}{2^3}\cdots+\frac{1}{2^{n-1}}=2-\frac{1}{2^{n-1}}.$  

Para:    $n=1$,        $\frac{1}{2^{1-1}}= 2-\frac{1}{2^{1-1}}$

                                $\frac{1}{2^0}= 2-\frac{1}{2^0}$ 
                                    
                                   $\frac{1}{1}= 2-\frac{1}{1}$
                              
                                       $1=1.$                            $\checkmark$



       $n=k$
HI           
$1+\frac{1}{2}+\frac{1}{2^2}+\frac{1}{2^3}\cdots+\frac{1}{2^{k-1}}=2-\frac{1}{2^{k-1}}.$
                                                                                       $\checkmark$

     $n=k^*$
TI
$1+\frac{1}{2}+\frac{1}{2^2}+\frac{1}{2^3}\cdots+\frac{1}{2^{k^*-1}}=2-\frac{1}{2^{k^*-1}}.$

Prueba:
$1+\frac{1}{2}+\frac{1}{2^2}+\frac{1}{2^3}\cdots+\frac{1}{2^{k^*-1}}=\underbrace{1+\frac{1}       {2}+\frac{1}{2^2}+\frac{1}{2^3}\cdots+\frac{1}{2^{k-1}}}+\frac{1}{2^{k^*-1}}$

                                                 $=\underbrace{2-\frac{1}{2^{k-1}}} +\frac{1}{2^{k^*-1}}$                                                                                        HI

                                                $=2-\frac{1}{2^k\cdot 2^{-1}} +\frac{1}{2^{\left(k+1\right)-1}}$

                                               $=2-\frac{2}{2^k} +\frac{1}{2^k}$

                                               $=2-\frac{1}{2^k} =2-\frac{1}{2^{\left(k+1\right)-1}}$

                                               $=2-\frac{1}{2^{k^*-1}}$              $\checkmark$

Ejercicio 7. Demostrar 
    
   $1^3+2^3+3^3+\cdots+n^3=\frac{n^2}{4}\left(n+1\right)^2.$

Demostración:

Para: $n=1$,                $1^3=\frac{1^2}{4}\left(1+1\right)^2$

                                        $1=\frac{1^2}{4}2^2$ 

                                        $1=\frac{1}{4}4$

                                                            $1=1$                         $\checkmark$
HI   
Para $n=k$,

$1^3+2^3+3^3+\cdots+k^3=\frac{k^2}{4}\left(k+1\right)^2.$    $\checkmark$

                                                                                    
TI
Para $n=k^*$,

$1^3+2^3+3^3+\cdots+\left(k^*\right)^3=\frac{\left(k^*\right)^2}{4}\left(k^*+1\right)^2.$

Demostración:
$1^3+2^3+3^3+\cdots+\left(k^*\right)^3=\underbrace{1^3+2^3+3^3+\cdots+k^3}+\left(k^*\right)^3$ 
                                                                $=\underbrace{\frac{k^2}{4}\left(k+1\right)^2}+\left(k+1\right)^3$
                                                               
                                                               $=\left(k+1\right)^2\left(\frac{k^2}{4}+k+1\right)$

                                                             $=\left(k+1\right)^2 \frac{\left(k^2+4k+4\right)}{4}$

                                                           $=\frac{\left(k+1\right)^2}{4}\left(k+2\right)^2$
                         
                                                          $=\frac{\left(k+1\right)^2}{4}\left(\left(k+1\right)+1\right)$

                                                                                       
                                                                                 $=\frac{\left(k^*\right)^2}{4}\left(k^*+1\right)^2.$          $\checkmark$
                                                                                                                                       


















































miércoles, 3 de diciembre de 2014

Ejercicios sobre Inducción Matemática I.

En  esta entrada se estudian ejercicios clásicos de inducción matemática.Se intenta dar claridad al desarrollo de los problemas. En todos los casos se toma     $\mathbb{N}=\{1, 2, 3, ... \}.$


Ejercicio 1. Demostrar que $1+2+3+\cdots +n= \frac{1}{2}n(n+1).$        $(Ec. 1)$

Demostración:
         Para $n=1$, tenemos   $1=\frac{1}{2}(1)(1+1)=\frac{1}{2}(1)(2)=1.$       $\checkmark$

Supongamos ahora que la proposición se cumple para $n=k$, es decir supongamos que la proposición
         
                       $1+2+3+\cdots +k = \frac{1}{2}k(k+1)$  es verdadera, esta es nuestra Hipótesis de Inducción, que indicamos como HI,  es decir, suponemos
                     
          HI             $1+2+3+\cdots +k = \frac{1}{2}k(k+1)$                 $\checkmark$  

Nos queda por probar  que

   $1+2+3+\cdots +k^* = \frac{1}{2}k^*(k^*+1)$    es verdadera, esta es nuestra Tesis de Inducción que indicamos como TI, es decir, nos falta por probar

  TI               $1+2+3+\cdots +k^* = \frac{1}{2}k^*(k^*+1)$              $\checkmark$.

Prueba:

  $1+2+3+\cdots +k^*=\underbrace{1+2+3+\cdots +k}+k^*=\underbrace{\frac{1}{2}k(k+1)}+(k+1)$
                                                          HI                                   HI

                                 $=\frac{k}{2}(k+1)+1\cdot(k+1)=(k+1)(\frac{k}{2}+1)=(k+1)(\frac{k+2}{2})$
                                 $=(k+1) \left( \frac{(k+1)+1}{2} \right)=\frac{1}{2}k^*(k^*+1)$.           $\checkmark$

Ya que es la misma $(Ec. 1)$ cuando sustituimos $n$ por $k^*$.

                           $\therefore 1+2+3+\cdots +n= \frac{1}{2}n(n+1).$


Ejercicio 2.  Demostrar  $ 2+4+6+\cdots +2n=n(n+1).$                           $(Ec. 2)$

Demostración:

           Para $n=1$, tenemos  $2\cdot 1=1\cdot (1+1)=1\cdot 2=2.$                $\checkmark$

 HI                   Para $n=k$, suponemos  $ 2+4+6+\cdots +2k=k(k+1).$                $\checkmark$

          Nos queda probar que

 TI                           Para $n=k^*$,   $ 2+4+6+\cdots +2k^*=k^*(k^*+1).$   

Prueba:

$ 2+4+6+\cdots +2k^*=2+4+6+\cdots +2k+2k^*= \underbrace{2+4+6+\cdots +2k}+2k^*$
                                                                                                             HI  

                                  $=\underbrace{k(k+1)}+2(k+1)=(k+1)(k+2)=k^*(k^*+1).$         $\checkmark$
                                            HI 


Ya que es la misma $(Ec. 2)$ cuando sustituimos $n$ por $k^*$.

                        $\therefore  2+4+6+\cdots +2n=n(n+1).$

Ejercicio 3. Demostrar que $3+6+9+\cdots +3n= \frac{3}{2}n(n+1).$        $(Ec. 3)$

Demostración:
         Para $n=1$, tenemos   $3\cdot 1=\frac{3}{2}(3)(1+1)=\frac{3}{2}(2)=3.$       $\checkmark$

Supongamos ahora que la proposición se cumple para $n=k$,
                   
          HI               $3+6+9+\cdots +3k = \frac{3}{2}k(k+1)$                 $\checkmark$

Nos queda por probar  que

  TI                  $3+6+9+\cdots +3k^* = \frac{3}{2}k^*(k^*+1)$              es verdadera.

Prueba:

 $3+6+9+\cdots +3k^*=\underbrace{3+6+9+\cdots 3k}+3k^*=\underbrace{\frac{3}{2}k(k+1)}+3(k+1)$
                                                        HI                                     HI

                                 $=3(k+1)\left( \frac{k}{2}+1\right)=3(k+1)\left(\frac{k+2}{2}\right))$
                                 $=\frac{3}{2}(k+1) \left( (k+1)+1 \right)=\frac{3}{2}k^*(k^*+1)$.           $\checkmark$

Ya que es la misma $(Ec. 3)$ cuando sustituimos $n$ por $k^*$.

                    $\therefore3+6+9+\cdots +3n= \frac{3}{2}n(n+1).$


     

jueves, 27 de noviembre de 2014

Postulados de Peano III.

OPERACIONES CON NÚMEROS NATURALES.

En esta Entrada se revisa la multiplicación de los números naturales aplicando los postulados de Peano, justificando sus diversas leyes: Ley de clausura, ley asociativa, ley conmutativa, ley distributiva,ley de cancelación.


Multiplicación sobre $\mathbb{N}$.
 La multiplicación se define por

 iii)   $n\cdot1=n$.
iv)    $n\cdot m^*=n\cdot m+n$    siempre que   $n\cdot m$   esté definido.

La multiplicación esta regida por las leyes:
                               
                                  $\forall m,n,p \in \mathbb{N}$,

$B_1$  Ley de clausura       $m\cdot n \in \mathbb{N}$

Demostración:
Sea la proposición  $P(n): m\cdot n \in \mathbb{N}$, para cualquier $m$ fijo y $n$ arbitrario.

Entonces  $P(1): m\cdot 1=m$ y tenemos que $P(1)$ es verdadera.

Supongamos  ahora  $P(k): m\cdot k$  verdadera.

Nos falta probar que $P(k^*)$ es verdadera.
Prueba:
                                 $P(k^*): m\cdot k^*= m\cdot k+m$  $\in \mathbb{N}$,
                                 ya que $m\cdot k \in \mathbb{N}$ por nuestra hipótesis de inducción y hemos supuesto $m \in \mathbb{N}$ desde el inicio.

                    $\therefore m\cdot n \in \mathbb{N}$ es verdadera.





Ejercicio preliminar : Demostrar que   $1\cdot n =n\cdot 1$
 Demostración:        Sea $P(n): 1\cdot n=n\cdot 1$
Tenemos que                  $P(1): 1\cdot 1=1\cdot 1$ es verdadera,
supongamos  ahora que $P(k): 1\cdot k=k\cdot 1$ es verdadera,
entonces      $P(k^*): 1\cdot k^*= 1\cdot k+1=k\cdot 1+1=k+1=k^*=k^*\cdot 1$ es verdadera.
                     $\therefore n\cdot 1=1\cdot n$ es verdadera.
                     $ \therefore n\cdot 1=1\cdot n=n$ es verdadera

$B_2$  Ley distributiva de la multiplicación sobre la suma por la derecha .                                                                             $ (m+p)\cdot n=m\cdot n+p\cdot n$.

Demostración: Sea   $P(n):  (m+p)\cdot n=m\cdot n+p\cdot n$.

Como    $P(1):  (m+p)\cdot 1=m+p=m\cdot 1+p\cdot 1$, tenemos que  $P(1)$ es verdadera,

supongamos ahora que  $P(k):(m+p)\cdot k=m\cdot k+p\cdot k$   es verdadera,

nos queda por probar que  $P(k^*)$  es verdadera.

Prueba:  $P(k^*):  (m+p)\cdot k^*=(m+p)\cdot k+(m+p)=m\cdot k+p\cdot k+m+p$

                                                  $ =(m\cdot k+m)+(p\cdot k+p)=m\cdot k^*+p\cdot k^*$,

y encontramos  que  $P(k^*)$   es verdadera,

                               $\therefore (m+p)\cdot n= m\cdot n+p\cdot n$  es verdadera.




$B_3$ Ley conmutativa   $n\cdot m=m\cdot n$.

Demostración:  Sea  $P(m): n\cdot m=m\cdot n$,  para cualquier  $n$ fijo y  $m$ arbitrario.

Tenemos que          $P(1): n\cdot 1=1\cdot n$  es verdadera ,

supongamos  ahora que  $P(k): n\cdot k=k\cdot n$  es verdadera,

Nos queda por probar que  $P(k^*): n\cdot k^*=k^*\cdot n$  es verdadera.

Prueba:   $P(k^*): n\cdot k^*=n\cdot k +n=k\cdot n+1\cdot n=(k+1)\cdot n=k^*\cdot n$  es verdadera.

                        $\therefore n\cdot m=m\cdot n$ es verdadera.






$B_4$   Ley distributiva del producto sobre la suma por la izquierda 

                     $m\cdot (n+p)=m\cdot n+ m\cdot p$.

Demostración:        Sea  $P(m): m\cdot (n+p)=m\cdot n+m\cdot p$.

Como         $P(1): 1\cdot (n+p)=n+p=1\cdot n+1\cdot p$  es verdadera,

podemos suponer que  $P(k): k\cdot (n+p)= k\cdot n+k\cdot p$  es verdadera.

Tenemos ahora que probar que  $P(k^*)$  es verdadera.

Prueba:
  $P(k^*): k^*\cdot (n+p)= (k+1)\cdot (n+p)=k\cdot(n+p)+1\cdot(n+p)$       (aplicar $B_2$)

                                                 $=k\cdot n+k\cdot p+n+p=(k\cdot n+n)+(k\cdot p+p)$

                                                 $=(k\cdot n+1\cdot n)+(k\cdot p+1\cdot p)$   (aplicar $B_2$)

                                                 $=(k+1)\cdot n+(k+1)\cdot p$

                                                 $=k^*\cdot n+k^*\cdot p$,  es verdadera.

                     $\therefore m\cdot (n+p)=m\cdot n+m\cdot p$  es verdadera.






$B_5$  Ley asociativa    $m\cdot (n\cdot p)=(m\cdot n)\cdot p.$

Demostración: Sea  $P(p): m\cdot (m\cdot p)=(m\cdot n)\cdot p.$

Como  $P(1): m\cdot(n\cdot 1)=m\cdot n=(m\cdot n) \cdot 1$  es verdadera,

podemos suponer que

          $P(k): m\cdot(n\cdot k)=(m\cdot n) \cdot k$  es verdadera.

Nos queda por probar que $P(k^*): m\cdot (n\cdot k^*)=(m\cdot n)\cdot k^*$ es verdadera.

Prueba:   $P(k^*):m\cdot (n\cdot k^*)=m\cdot (n \cdot k+n)$.      (aplicar $B_4$)
                                                         $=m\cdot (n\cdot k)+m\cdot n$
                                                         $=(m\cdot n)\cdot k+(m\cdot n)\cdot 1$
                                                         $=(m\cdot n)(k+1)=(m\cdot n)\cdot k^*$ es verdadera.

             $\therefore m\cdot (n\cdot p)=(m\cdot n)\cdot p$.


$B_6$  Ley de cancelación  Si  $m\cdot p=n\cdot p$,  entonces  $m=n$.

Demostración: Sea  $P(p)$:   $m\cdot p=n\cdot p$.

Como   $P(1)$ es     $m\cdot 1=n\cdot 1$,  entonces  $m=n$,

tenemos que $P(1)$  es verdadera.

Supongamos ahora que  $P(k)$  es verdadera, es decir, supongamos que

                         $m\cdot k=n\cdot k$.

Nos queda por probar que  $P(k^*)$: Si $m\cdot k^*=n\cdot k^*$,  entonces  $m=n$ .

Prueba:
                                Si  $m\cdot k^*=n\cdot k^*$,  entonces   $m\cdot k+m=n\cdot k+n.$ .

                                Si  $m\cdot k+m=n\cdot k+n$, entonces $m=n$  es verdadera,

                                              ya que si   $m\cdot k=n\cdot k$  podemos aplicar $A_4$
                             
                              $\therefore$ Si $m\cdot p=n\cdot p$, entonces $m=n$ .


                             




















sábado, 22 de noviembre de 2014

Postulados de Peano II.

Operaciones con números naturales.

Adición sobre $\mathbb{N}$.

i)    $n+1=n^*$     $\forall n \in \mathbb{N}$.
ii)  $n+m^*=(n+m)^*$  siempre que $n+m$ esté definido.

La adición está regida por la siguientes leyes:

             $\forall$ $ m,n,p \in \mathbb{N}$,

$A_1$. Ley de clausura:   $n+m$  $\in$  $\mathbb{N}.$

Demostración:
Sea la proposición
                    $P(m):$     $n+m$ $\in$  $\mathbb{N}$.

Entonces:     $P(1):$      $n+1$  $\in$   $\mathbb{N}$.    (Postulado II)

Supongamos ahora que para algún $k$ $\in$ $\mathbb{N}$

                     $P(k):$      $n+k$  $\in$   $\mathbb{N}$ es cierto.

Como           $P(k^*):$      $n+k^*=(n+k)^*$  $\in$   $\mathbb{N}$ es cierto.   (Postulado II)

Tenemos que $P(m)$ es válido $\forall$ $m$ $\in$  $\mathbb{N}$,

y como $n$ $\in$ $\mathbb{N}$ es arbitrario concluimos que $A_1$ es válida $\forall$ $m,n$ $\in$  $\mathbb{N}.$

$A_2$. Ley asociativa:   $n+(m+p)=(n+m)+p$.
Demostración:
Sea la proposición     $P(p): n+(m+p)=(n+m)+p$, para cualquier $p \in \mathbb{N}$, con $m,n, \in \mathbb{N}$ arbitrarios. 

Entonces, para $n=1$ tenemos
                   $P(1): n+(m+1)=n+m^*=(n+m)^*=(n+m)+1$ , y vemos que $P(1)$ es cierto.

Ahora supongamos que $P(k)$ es cierto para algún $k \in \mathbb{N}$, es decir, suponemos que   
                                   $P(k): n+(m+k)=(n+m)+k$            es cierto.
Nos queda ahora por probar que
                       $P(k^*): n+(m+k^*)=(n+m)+k^*$ es verdadero.

Prueba:
                                   $P(k^*): n+(m+k^*)=n+(m+k)^*=[n+(m+k)]^*$,

pero $P(k)$ nos permite sustituir $n+(m+k)$ por $(n+m)+k$, entonces $P(k^*)$ toma la forma

          $P(k^*): n+(m+k^*)=n+(m+k)^*=[n+(m+k)]^*=[(n+m)+k]^*=(n+m)+k^*$.

Tenemos de esta manera que $P(p)$ es válida y como $m$ y $n$ son arbitrarios,

                              $n+(m+p)=(n+m)+p$

es válida par todos los números naturales.
   
Ejercicio preliminar: Demostrar que           $n+1=1+n$.
Demostración:  Sea                               $P(n): n+1=1+n$,
Entonces:                                             $P(1): 1+1=1+1,$ es verdadera.
Supongamos que para algún                 $k \in\mathbb{N}$,
                                                           $P(k): k+1=1+k$ es verdadera.
Tenemos  que                            $P(k^*): k^*+1=(k+1)+1=(1+k)+1=1+(k+1)=1+k^*$ es verdadera.
                                                         $\therefore n+1=1+n$.

$A_3$. Ley conmutativa:  $n+m=m+n$.

Demostración:   Sea $P(m): n+m=m+n$.

Entonces     $P(1): n+1=1+n$ es verdadera. 
        
Supongamos que para algún       $k \in\mathbb{N}$,
                  $P(k): n+k=k+n$  es verdadera.

Tenemos  que      $P(k^*): n+k^*=(n+k)^*=(k+n)^*=k+n^*$ 
                                                  $=k+(n+1)=k+(1+n)=(k+1)+n=k^*+n$ es verdadera,
                                                         $\therefore n+m=m+n$.

$A_4$. Ley de cancelación:  Si $m+p=n+p$, entonces $m=n$.

Demostración:   Hagamos $P(p)$:  Si $m+p=n+p$, entonces $m=n$.

Sea      $P(1)$: Si  $m+1=n+1$, entonces $m=n$,

entonces $P(1)$ la podemos escribir como

            $P(1)$: Si $m^*=n^*$, entonces $m=n$  (Postulado IV),

y tenemos que $P(1)$ es verdadera.

Ahora supongamos que 
                    $P(k)$: Si $m+k=n+k$ , entonces $m=n$ es verdadera.

Nos queda por demostrar que $P(k^*)$ es verdadera.

Demostración:

Tengamos  
                   $P(k^*)$: Si $m+k^*=n+k^*$, entonces $(m+k)^*=(n+k)^*$,
                               
                                 Si $(m+k)^*=(n+k)^*$, entonces $m+k=n+k$,

                                Si $m+k=n+k$, entonces $m=n$.

Y encontramos que $P(k^*)$ es verdadera,

                 $\therefore $ Si $m+p=n+p$, entonces $m=n$, es verdadera.






miércoles, 19 de noviembre de 2014

Postulados de Peano I

Postulados de Peano.

Sea un conjunto no vacío $\mathbb{N}$ tal que
Postulado I: $1 \in \mathbb{N}$.
Postulado II: $ \ \forall n  \in  \mathbb{N},  \ \exists$ un único $n^* \ \in \ \mathbb{N}$, llamado siguiente de $n$.
Postulado III:$ \ \forall n \in \mathbb{N}$ se tiene $n^* \neq 1$.
Postulado IV: Si $m,\ n \in \mathbb{N}$ y $m^* = n^*$, entonces $m=n$.
Postulado V: Todo subconjunto $K$ de $\mathbb{N}$ que tenga las propiedades
                                  a)  $1\in K$
                                  b)  $k^* \in K$ siempre que $k \in K$
                       es el mismo $\mathbb{N}$.
Nota. El establecimiento del Postulado II nos permite definir $n^*=n+1, \forall n\in \mathbb{N}$.

Nos damos cuenta a partir de los Postulados de I y III que el número 1  es el primer elemento del conjunto $\mathbb{N}$, el conjunto de los números naturales, que podemos visualizar de la siguiente manera:

                $\mathbb{N}$={1, $1^*$, $1^{**}$, $1^{***}$, $\cdots$}={1,2,3,4, $\cdots$}


Ejercicio. Utilizando los postulados de Peano vamos a demostrar que

                                  $ m^* \neq m, \forall m \in \mathbb{N}$.

Entonces, sea la proposición

                                $P(m): m^* \neq m, \forall m \in \mathbb{N}$.

Con esto queremos decir que nuestra variable $m$, va tomar todos sus valores de $\mathbb{N}$.

Así tenemos,
                             $P(1): 1^* \neq 1$  es verdadero por el Postulado III,

                             $P(2): 2^* \neq 2$ es verdadero porque $2^*$ es el siguiente de 2,
                                        es decir, $2^*=3 \neq 2$,

                              $P(3): 3^* \neq 3$ es verdadero porque $3^*$ es el siguiente de 3,
                                        es decir, $3^*=4 \neq 3$,

encontramos que $P(m)$ se cumple para los primeros valores de $\mathbb{N}$, sin embargo, ¿se cumplirá para valores mayores?,  ¿se cumplirá para $n=1000$?, ¿para $n=10 000$?, ¿para $n=50 000$?.
Junto con la imposibilidad de revisar uno por uno todos los valores, nos topamos también con el hecho de que por grande que sea el número que hayamos revisado, siempre hay uno más grande por revisar.
Para intentar una solución  definimos un conjunto $A$ de la siguiente manera:

                                            $A$={x: x $\in$  $\mathbb{N}$, $P(k)$ es verdadero}.
Como
                             $1\in \mathbb{N}$                                          (Postulado I)

y                           $P(1)$ es cierto                               (Postulado III),

entonces,                           $ 1\in A$.

Sea ahora  $k$ cualquier elemento de $A$,
tenemos  que    
                                        $k \in \mathbb{N}$  y  $P(k)$ es verdadero.
Es decir,
                                        $k \in \mathbb{N}$  y  $ k^* \neq k.$

Si $k \in\mathbb{N}$, entonces $k^*\in \mathbb{N}.$                    (Postulado II)

Si $k^* \neq k$, entonces $(k^*)^*\neq(k^*)$,

y tenemos que

                             $P(k^*): (k^*)^* \neq (k^*)$ es verdadero.

Hemos encontrado que

                                   $k^* \in \mathbb{N}$  y  $P(k^*)$ es verdadero,

entonces

                                        $k^* \in A$.
Como

            $1\in A$ y   $k^* \in A$ siempre que $k \in A$,

entonces     $A=\mathbb{N}$.                                                   (Postulado V)


       $\therefore$  $P(m): m^* \neq m, \forall m \in \mathbb{N}$ es válida $\forall m\in \mathbb{N}.$

Podemos hacer dos anotaciones con respecto del Ejercicio anterior:
-Primera: no necesitamos revisar $P(m)$ para cada uno de los valores de $\mathbb{N}$, puesto que $k$ es un elemento cualquiera de $\mathbb{N}$ y sin embargo, como vimos, $P(m)$ se cumple $\forall n \in \mathbb{N}$,
-Segunda: por grande que sea $m \in \mathbb{N}$,  hemos demostrado que $P(m)$ se cumple para el siguiente de $m$.

Al establecer la validez de la proposición anterior, hemos probado al mismo tiempo el

Principio de inducción matemática:

Una proposición $P(m)$ es cierta $\forall m \in \mathbb{N}$ siempre que:

                                                     $P(1)$ sea cierto

y que $\forall   k  \in \mathbb{N}$,       $P(k)$ cierto implique  $P(k^*)$ cierto.
                 






jueves, 2 de octubre de 2014

Inducción matemática. Ejemplo 4.

Inducción matemática.

                                                                                                                                                                Ejemplo 4. Demostrar que $x+y$ divide a $\left(x^{2n}-y^{2n}\right)$.



Entonces, sea

$P(n):(x+y)\mid \left (x^{2n}-y^{2n}\right)$,   $ \forall\ {n} \in \mathbb{N}$                                                     $(2.1)$

la proposición que vamos a demostrar.


Solución:
Primero probamos la proposiciónn $(2.1)$ para $n=1$.



         $ P(1):\frac{x^{2(1)}-y^{2(1)}}{x+y}$$=\frac{x^{2}-y^{2}}{x+y}$
                                                                           $=\frac{(x+y)(x-y)}{x+y}$
                                                                           $= x-y$
                                                                       


vemos que la proposiónn $(2.1)$  sí se cumple para $n=1$.

Ahora establecemos nuestra hipótesis de inducción afirmando que


 $P(k):(x+y)\mid \left (x^{2(k)}-y^{2(k)}\right)$                                                  $(2.2)$

es cierta.

Para terminar el problema vamos a demostrar que


$ P(k+1):(x+y)\mid \left (x^{2(k+1)}-y^{2(k+1)}\right)$                                  $(2.3)$


es cierta.


Tenemos entonces que:


$\frac{x^{2(k+1)}-y^{2(k+1)}}{x+y}$=$\frac{x^{2k+2}-y^{2k+2}}{x+y}$
                                 

Reacomodando el último cociente obtenemos


 $ \frac{x^{2k+2}-y^{2k+2}}{x+y}$ = $\frac{x^{2k+2}-x^{2k+1}y+x^{2k+1}y-x^{2k}y^2+x^{2k}y^2-y^{2k+2}}{x+y}$                              $(2.4)$                                                                    
                               $=\frac{x^{2k+1+1}-x^{2k+1}y+x^{2k+1}y-x^{2k}y^2+x^{2k}y^2-y^{2k+1+1}}{x+y}$
                               $=\frac{x^{2k+1}x-x^{2k}xy+x^{2k+1}y-x^{2k}y^2+x^{2k}y^2-y^{2k}y^2}{x+y}$
                              $=\frac{x^{2k+1}(x-y)-x^{2k}y(x+y)+y^2(x^{2k}-y^{2k})}{x+y}$          
                               $=x^{2k+1}-x^{2k} y+y^2  \frac{x^{2k}-y^{2k}}{x+y}.$        $(2.5)$




Proposición que es exactamente divisible entre $x+y$, ya que el segundo factor del último término de la ec.$(2.5)$ es nuestra hipótesis de inducción $(2.2)$, con lo cual terminamos nuestra demostración.


Puede parecer arbitrario el arreglo que hicimos para darle forma a la ec.$(2.4)$, sin embargo no lo es en absoluto ya que si  operamos el cociente implicado por la ec.$(2.3)$ obtenemos, dependiendo de si tomamos uno, dos,tres o los términos que queramos en el cociente de la división:

a) Si tomamos el primer término del cociente obtenemos;




                                    $x^{2k+1}$          
           _____________________________________________
 $x+y\mid$    $x^{2k+2}$                                      $-y^{2k+2}$
              $-x^{2k+2}$     $-x^{2k+1}y$        
               ___________________________________________________                                
                                $-x^{2k+1}y$                    $-y^{2k+2}$ .



 Aplicando el algoritmo de la división a la última expresión tenemos:




$\frac{x^{2k+2}-y^{2k+2}}{x+y}$  =  $x^{2k+1}-\frac{x^{2k+1}y+y^{2k+2}}{x+y}$
                             = $\frac{x^{2k+1}\left(x+y\right)-\left(x^{2k+1}y+y^{2k+2}\right)}{x+y}$
                             = $\frac{x^{2k+2}+x^{2k+1}y-x^{2k+1}y-y^{2k+2}}{x+y}$
                             = $\frac{x^{2k+2}-y^{2k+2}}{x+y}.$



b) Si tomamos los dos primeros términos del cociente nos queda:






                                 $x^{2k+1}$     $-x^{2k}y$                                                                                                  ________________________________________________________
  $x+y$ $\mid$  $x^{2k+2}$                                                       $-y^{2k+2}$
             $-x^{2k+2}$  $-x^{2k+1}y$                                
            __________________________________________________________
                            $-x^{2k+1}y$                                          
                            $+x^{2k+1}y$     $+x^{2k}y^2$  
                            __________________________________________
                                             $+x^{2k}y^2$                  $-y^{2k+2}$



y utilizando el algoritmo de la división, encontramos:




$\frac{x^{2k+2}-y^{2k+2}}{x+y}$  $=x^{2k+1}-x^{2k}y+\frac{x^{2k}y^2-y^{2k+2}}{x+y}$                        $(2.6)$
                              $=\frac{\left(x^{2k+1}-x^{2k}y\right)(x+y)+x^{2k}y^2-y^{2k+2}}{x+y}$
                              $=\frac{x^{2k+2}-x^{2k+1}y+x^{2k+1}y-x^{2k}y^2+x^{2k}y^2-y^{2k+2}}{x+y}$              $(2.7)$
                              $=\frac{x^{2k+2}-y^{2k+2}}{x-y}.$




c) Si tomamos los tres primeros términos del cociente encontramos:


                 
                                                      $x^{2k+1}-x^{2k}y+x^{2k-1}y^2$
                                  _____________________________________________________
                       $x+y \mid  \ \ x^{2k+2}$                                                             $- y^{2k+2}$
                                  $-x^{2k+2}-x^{2k+1}y$
                                       _________________
                                                 $-x^{2k+1}y$
                                                 $+x^{2k+1}y+x^{2k}y^2$
                                                  _____________________
                                                                  $+x^{2k}y^2$
                                                                  $-x^{2k}y^2-x^{2k-1}y^3$
                                                           ____________________________________
                                                                                 $-x^{2k-1}y^3       \ \ \   - y^{2k+2}.$

Si hacemos lo mismo que en los dos casos anteriores llegamos a la expresión:


$\frac{x^{2k+2}-y^{2k+2}}{x+y}$  $=x^{2k+1}-x^{2k}y+x^{2k-1}y^2-\frac{x^{2k-1}y^3+y^{2k+2}}{x+y}$
                              $=\frac{\left(x^{2k+1}-x^{2k}y+x^{2k-1}y^2\right)(x+y)-x^{2k-1}y^3-y^{2k+2}}{x+y}$
                              $=\frac{x^{2k+2}-x^{2k+1}y+x^{2k}y^2+x^{2k+1}y-x^{2k}y^2+x^{2k-1}y^3-x^{2k-1}y^3-y^{2k+2}}{x+y}$
                              $=\frac{x^{2k+2}-y^{2k+2}}{x+y}.$


Advertimos que el arreglo que hicimos para solucionar la última parte de nuestro problema de inducción fue comenzar con la ec.$(2.7)$ y terminar con la ec.$(2.6)$, ya que ésta última también se puede escribir en la forma
                                    $x^{2k+1}-x^{2k} y+y^2 \    \frac{x^{2k}-y^{2k}}{x+y}.$

Concluimos nuestro ejercicio revisando la naturaleza del cociente
                                  $\frac{x^{2n}-y^{2n}}{x+y}.$
A partir de la ec. $(2.2)$ tenemos que el cociente $\frac{x^{2k}-y^{2k}}{x+y}$ es una fracción entera, y que por lo tanto su residuo es cero. También sabemos que en la división de dos polinomios homogéneos en la que el grado del dividendo es $2k$ y el grado del divisor es $1$, el resultado  es un polinomio homogéneo de grado $2k-1$.
De acuerdo a lo anterior tenemos;


                                $x^{2k-1}-x^{2k-2}y+$    $\cdots$                               $+xy^{2k-2}-y^{2k-1}$
             _________________________________________________________________________
$x+y \mid  x^{2k}$                                                                                                              $ -y^{2k}$
            $-x^{2k}-x^{2k-1}y$
             ___________________
                       $-x^{2k-1}y$
                       $+x^{2k-1}y+x^{2k-2}y^2$
                            ________________________
                                          $+x^{2k-2}y^2$
                                                                            $\cdots$
                                                                                            $+x^2y^{2k-2}$
                                                                                            $-x^2y^{2k-2}-xy^{2k-1}$
                                                                                           _________________________
                                                                                                                   $-xy^{2k-1}$
                                                                                                                   $+xy^{2k-1}+y^{2k}$
                                                                                                                  ________________
                                                                                                                          $0$         $0.$
                                         
                                       
                                                                 

                                                                     
Al sustituir este resultado en la ec.$(2.5)$ nos queda:


$ \frac{x^{2k+2}-y^{2k+2}}{x-y}$  $=x^{2k+1}-x^{2k}y+y^2  \frac{x^{2k}-y^{2k}}{x+y}$                              
                               $=x^{2k+1}-x^{2k}y+y^2\left(x^{2k-1}-x^{2k-2}y+\cdots+xy^{2k-2}-y^{2k-1}\right)$
                               $=x^{2k+1}-x^{2k}y+x^{2k-1}y^2-x^{2k-2}y^3+\cdots+xy^{2k}-y^{2k+1}$ $(2.8).$

 Como $k \ \in \mathbb{N}$, la ec.$(2.8)$ toma la forma

   $\frac{x^{2n}-y^{2n}}{x+y}=x^{2n-1}-x^{2n-2}y+x^{2n-3}y^2+\cdots+xy^{2n-2}-y^{2n-1},\  \  \forall{n} \in \mathbb{N}.$     $ (2.9)$







miércoles, 1 de octubre de 2014

Inducción matemática. Ejemplo 3.

Inducción matemática.

                                                                                                                                                                Ejemplo 3. Demostrar que $x-y$ divide a $\left(x^{2n}-y^{2n}\right)$.



Entonces, sea

$P(n):(x-y)\mid \left (x^{2n}-y^{2n}\right)$,   $ \forall\ {n} \in \mathbb{N}$                                                     $(2.1)$

la proposición que vamos a demostrar.


Solución:
Primero probamos la proposiciónn $(2.1)$ para $n=1$.



         $ P(1):\frac{x^{2(1)}-y^{2(1)}}{x-y}$$=\frac{x^{2}-y^{2}}{x-y}$
                                                                           $=\frac{(x+y)(x-y)}{x-y}$
                                                                           $= x+y$
                                                                         


vemos que la proposiónn $(2.1)$  sí se cumple para $n=1$.

Ahora establecemos nuestra hipótesis de inducción afirmando que


 $P(k):(x-y)\mid \left (x^{2(k)}-y^{2(k)}\right)$                                                  $(2.2)$

es cierta.

Para terminar el problema vamos a demostrar que


$ P(k+1):(x-y)\mid \left (x^{2(k+1)}-y^{2(k+1)}\right)$                                  $(2.3)$


es cierta.


Tenemos entonces que:


$\frac{x^{2(k+1)}-y^{2(k+1)}}{x-y}$=$\frac{x^{2k+2}-y^{2k+2}}{x-y}$
                                   

Reacomodando el último cociente obtenemos


 $ \frac{x^{2k+2}-y^{2k+2}}{x-y}$ = $\frac{x^{2k+2}+x^{2k+1}y-x^{2k+1}y-x^{2k}y^2+x^{2k}y^2-y^{2k+2}}{x-y}$                              $(2.4)$                                                                      
                               $=\frac{x^{2k+1+1}+x^{2k+1}y-x^{2k+1}y-x^{2k}y^2+x^{2k}y^2-y^{2k+1+1}}{x-y}$
                               $=\frac{x^{2k+1}x+x^{2k}xy-x^{2k+1}y-x^{2k}y^2+x^{2k}y^2-y^{2k}y^2}{x-y}$
                              $=\frac{x^{2k+1}(x-y)+x^{2k}y(x-y)+y^2(x^{2k}-y^{2k})}{x-y}$            
                               $=x^{2k+1}+x^{2k} y+y^2  \frac{x^{2k}-y^{2k}}{x-y}.$        $(2.5)$




Proposición que es exactamente divisible entre $x-y$, ya que el segundo factor del último término de la ec.$(2.5)$ es nuestra hipótesis de inducción $(2.2)$, con lo cual terminamos nuestra demostración.


Puede parecer arbitrario el arreglo que hicimos para darle forma a la ec.$(2.4)$, sin embargo no lo es en absoluto ya que si  operamos el cociente implicado por la ec.$(2.3)$ obtenemos, dependiendo de si tomamos uno, dos,tres o los términos que queramos en el cociente de la división:

a) Si tomamos el primer término del cociente obtenemos;




                                    $x^{2k+1}$            
           _____________________________________________  
 $x-y\mid$    $x^{2k+2}$                                      $-y^{2k+2}$  
              $-x^{2k+2}$     $+x^{2k+1}y$          
               ___________________________________________________                                  
                                $+x^{2k+1}y$                    $-y^{2k+2}$ .



 Aplicando el algoritmo de la división a la última expresión tenemos:




$\frac{x^{2k+2}-y^{2k+2}}{x-y}$  =  $x^{2k+1}+\frac{x^{2k+1}y-y^{2k+2}}{x-y}$
                             = $\frac{x^{2k+1}\left(x-y\right)+\left(x^{2k+1}y-y^{2k+2}\right)}{x-y}$
                             = $\frac{x^{2k+2}-x^{2k+1}y+x^{2k+1}y-y^{2k+2}}{x-y}$
                             = $\frac{x^{2k+2}-y^{2k+2}}{x-y}.$



b) Si tomamos los dos primeros términos del cociente nos queda:






                                 $x^{2k+1}$     $+x^{2k}y$                                                                                                  ________________________________________________________
  $x-y$ $\mid$  $x^{2k+2}$                                                       $-y^{2k+2}$
             $-x^{2k+2}$  $+x^{2k+1}y$                                  
            __________________________________________________________
                            $+x^{2k+1}y$                                            
                            $-x^{2k+1}y$     $+x^{2k}y^2$    
                                    __________________________________________
                                             $+x^{2k}y^2$                  $-y^{2k+2}$



y utilizando el algoritmo de la división, encontramos:




$\frac{x^{2k+2}-y^{2k+2}}{x-y}$  $=x^{2k+1}+x^{2k}y+\frac{x^{2k}y^2-y^{2k+2}}{x-y}$                        $(2.6)$
                              $=\frac{\left(x^{2k+1}+x^{2k}y\right)(x-y)+x^{2k}y^2-y^{2k+2}}{x-y}$
                              $=\frac{x^{2k+2}+x^{2k+1}y-x^{2k+1}y-x^{2k}y^2+x^{2k}y^2-y^{2k+2}}{x+y}$              $(2.7)$
                              $=\frac{x^{2k+2}-y^{2k+2}}{x-y}.$




c) Si tomamos los tres primeros términos del cociente encontramos:


                   
                                                      $x^{2k+1}+x^{2k}y+x^{2k-1}y^2$
                                  _____________________________________________________
                       $x-y \mid  \ \ x^{2k+2}$                                                             $- y^{2k+2}$
                                  $-x^{2k+2}+x^{2k+1}y$
                                       _________________
                                                 $+x^{2k+1}y$
                                                 $-x^{2k+1}y+x^{2k}y^2$
                                                       _____________________
                                                                  $+x^{2k}y^2$
                                                                  $-x^{2k}y^2+x^{2k-1}y^3$
                                                           ____________________________________
                                                                                 $+x^{2k-1}y^3       \ \ \   - y^{2k+2}.$

Si hacemos lo mismo que en los dos casos anteriores llegamos a la expresión:


$\frac{x^{2k+2}-y^{2k+2}}{x-y}$  $=x^{2k+1}+x^{2k}y+x^{2k-1}y^2+\frac{x^{2k-1}y^3-y^{2k+2}}{x-y}$
                              $=\frac{\left(x^{2k+1}+x^{2k}y+x^{2k-1}y^2\right)(x-y)+x^{2k-1}y^3-y^{2k+2}}{x-y}$
                              $=\frac{x^{2k+2}+x^{2k+1}y+x^{2k}y^2-x^{2k+1}y-x^{2k}y^2-x^{2k-1}y^3+x^{2k-1}y^3-y^{2k+2}}{x-y}$
                              $=\frac{x^{2k+2}-y^{2k+2}}{x-y}.$


Advertimos que el arreglo que hicimos para solucionar la última parte de nuestro problema de inducción fue comenzar con la ec.$(2.7)$ y terminar con la ec.$(2.6)$, ya que ésta última también se puede escribir en la forma
                                    $x^{2k+1}+x^{2k} y+y^2 \    \frac{x^{2k}-y^{2k}}{x-y}.$

Concluimos nuestro ejercicio revisando la naturaleza del cociente
                                  $\frac{x^{2n}-y^{2n}}{x-y}.$
A partir de la ec. $(2.2)$ tenemos que el cociente $\frac{x^{2k}-y^{2k}}{x-y}$ es una fracción entera, y que por lo tanto su residuo es cero. También sabemos que en la división de dos polinomios homogéneos en la que el grado del dividendo es $2k$ y el grado del divisor es $1$, el resultado  es un polinomio homogéneo de grado $2k-1$.
De acuerdo a lo anterior tenemos;


                                $x^{2k-1}+x^{2k-2}y+$    $\cdots$                               $+xy^{2k-2}+y^{2k-1}$
             _________________________________________________________________________
$x-y \mid  x^{2k}$                                                                                                              $ -y^{2k}$
            $-x^{2k}+x^{2k-1}y$
             ___________________
                       $+x^{2k-1}y$
                       $-x^{2k-1}y+x^{2k-2}y^2$
                            ________________________
                                          $+x^{2k-2}y^2$
                                                                            $\cdots$
                                                                                            $+x^2y^{2k-2}$
                                                                                            $-x^2y^{2k-2}+xy^{2k-1}$
                                                                                           _________________________
                                                                                                                   $+xy^{2k-1}$
                                                                                                                   $-xy^{2k-1}+y^{2k}$
                                                                                                                  ________________
                                                                                                                          $0$         $0.$
                                           
                                         
                                                                   

                                                                       
Al sustituir este resultado en la ec.$(2.5)$ nos queda:


$ \frac{x^{2k+2}-y^{2k+2}}{x-y}$  $=x^{2k+1}+x^{2k}y+y^2  \frac{x^{2k}-y^{2k}}{x-y}$                                
                               $=x^{2k+1}+x^{2k}y+y^2\left(x^{2k-1}+x^{2k-2}y+\cdots+xy^{2k-2}+y^{2k-1}\right)$
                               $=x^{2k+1}+x^{2k}y+x^{2k-1}y^2+x^{2k-2}y^3+\cdots+xy^{2k}+y^{2k+1}$ $(2.8).$

 Como $k \ \in \mathbb{N}$, la ec.$(2.8)$ toma la forma

   $\frac{x^{2n}-y^{2n}}{x-y}=x^{2n-1}+x^{2n-2}y+x^{2n-3}y^2+\cdots+xy^{2n-2}+y^{2n-1},\  \  \forall{n} \in \mathbb{N}.$     $ (2.9)$